Solving Eigenvector Problems: A+B and AB with Corresponding Eigenvalues

  • Thread starter Thread starter Jennifer1990
  • Start date Start date
  • Tags Tags
    Eigenvector
Click For Summary
The discussion focuses on proving that if v is an eigenvector of matrices A and B with eigenvalues lambda and mui, then v is also an eigenvector of A+B and AB. The user suggests adding the equations Av = lambda*v and Bv = mui*v to derive (A+B)v = (lambda + mui)v, confirming that v is an eigenvector of A+B with eigenvalue lambda + mui. For the product AB, the user notes that multiplying AB by v allows the eigenvalues to be treated as scalars, leading to the conclusion that v is an eigenvector of AB as well. The conversation emphasizes the importance of linearity and associativity in these operations. Overall, the thread provides a clear approach to solving eigenvector problems involving matrix sums and products.
Jennifer1990
Messages
55
Reaction score
0

Homework Statement


Suppose that v is an eigenvector of both A and B with corresponding eigenvalues lambda and mui respectively. Show that v is an eigenvector of A+B and of AB and determine the corresponding eigenvalues


Homework Equations





The Attempt at a Solution


Av = lambda*v
Bv = mui*v
this is all i can think of...can someone give me a hint abt the next step?
 
Physics news on Phys.org


Add the two equations together, tada! Of course, you'll need to exploit associativity... or linearity... man I am always getting terms confused.
 
Last edited:


u mean like:
(Av +Bv) = lambda*v + mui*v
(A+B)v = (lambda + mui) v
 


Yup! As for finding the eigenvalues of AB, simply multiply AB by v and remember that your eigenvalues are scalars that can move freely.
 
Question: A clock's minute hand has length 4 and its hour hand has length 3. What is the distance between the tips at the moment when it is increasing most rapidly?(Putnam Exam Question) Answer: Making assumption that both the hands moves at constant angular velocities, the answer is ## \sqrt{7} .## But don't you think this assumption is somewhat doubtful and wrong?

Similar threads

Replies
5
Views
2K
Replies
11
Views
2K
Replies
9
Views
2K
  • · Replies 2 ·
Replies
2
Views
2K
  • · Replies 5 ·
Replies
5
Views
2K
  • · Replies 7 ·
Replies
7
Views
1K
  • · Replies 3 ·
Replies
3
Views
1K
  • · Replies 1 ·
Replies
1
Views
5K
  • · Replies 5 ·
Replies
5
Views
14K
  • · Replies 3 ·
Replies
3
Views
2K